Revision history of "2018 OIM Problems/Problem 2"

Diff selection: Mark the radio boxes of the revisions to compare and hit enter or the button at the bottom.
Legend: (cur) = difference with latest revision, (prev) = difference with preceding revision, m = minor edit.

  • (cur | prev) 13:19, 14 December 2023Tomasdiaz (talk | contribs). . (633 bytes) (+633). . (Created page with "== Problem == Let <math>ABC</math> be a triangle such that <math>\angle BAC = 90^{\circ}</math> and <math>BA = CA</math>. Let <math>M</math> be the midpoint of <math>BC</math>...")